LSAT and Law School Admissions Forum

Get expert LSAT preparation and law school admissions advice from PowerScore Test Preparation.

 Administrator
PowerScore Staff
  • PowerScore Staff
  • Posts: 8917
  • Joined: Feb 02, 2011
|
#41386
Please post your questions below!
User avatar
 schmerg
  • Posts: 1
  • Joined: Jun 07, 2021
|
#87664
Answer is B. Management must be at 1 pm because Management has to come before Personnel.
User avatar
 Stephanie Oswalt
PowerScore Staff
  • PowerScore Staff
  • Posts: 811
  • Joined: Jan 11, 2016
|
#87696
schmerg wrote: Mon Jun 07, 2021 4:07 pm Answer is B. Management must be at 1 pm because Management has to come before Personnel.
Hi schmerg!

Thanks for the post! The answer to #2 is actually A. Please let us know if you have any questions regarding this question and answer choice. :) Thanks!
 g_lawyered
  • Posts: 211
  • Joined: Sep 14, 2020
|
#92858
Hi P.S.,
I rushed through this question I got it incorrect. I want to make sure I understand why answer choice E is incorrect. Is this the only hypothetical that makes answer choice E as CBT? That O and U can be at the same time...

Here's the hypothetical I drew:
In order of (1-3)
W: M, O, S
E: G, U, P
 Robert Carroll
PowerScore Staff
  • PowerScore Staff
  • Posts: 1787
  • Joined: Dec 06, 2013
|
#92874
GGIBA003,

You've switched or ignored the last rule - U must be earlier than G. So that hypothetical does not work. However, this should (in order):

West: SOG
East MUP

Robert Carroll
 g_lawyered
  • Posts: 211
  • Joined: Sep 14, 2020
|
#92920
Oops. I see my mistake now. Is this hypothetical that makes answer choice A correct (MBF)?
(A)- G earlier than M
W: S, M, O/P
E: G. U, P/O

Answer A also breaks rule 4?
Thanks in advance!
 Adam Tyson
PowerScore Staff
  • PowerScore Staff
  • Posts: 5153
  • Joined: Apr 14, 2011
|
#92997
I wouldn't suggest using hypothetical solutions to prove that an answer cannot be true, although you could use them to prove that some things could be true. For example, if you had answer B as a contender, you might try drawing out a solution in which P was earlier than G. That would mean we would have a sequence of M :longline: P :longline: G, which would place M at 1:00, P at 2:00, and G at 3:00. That could look like this:

West: M P S
East: U O G

(There may be other solutions that would also work, but all we have to do to prove that an answer to a Cannot Be True question is wrong is to show that it Could Be True in at least one case.)

For answer A, my suggestion would be to diagram the answer and see what it looks like. If we try to put G before M, when we couple that with the rule about U and G we get this sequence:

U :longline: G :longline: M

This cannot be true because M must be before both O and P, and this sequence makes that impossible by placing M at 3:00. There's no need for a hypothetical solution here, and no hypothetical solution would work in any case. Just a quick diagram of the answer choice in connection with the original rules reveals that it cannot be true and is therefore correct.
 g_lawyered
  • Posts: 211
  • Joined: Sep 14, 2020
|
#93055
Hi Adam,
Thanks for that explanation. I hadn't thought of using that approach to MBF questions. I'll start using it now, thank you.

Get the most out of your LSAT Prep Plus subscription.

Analyze and track your performance with our Testing and Analytics Package.